You can Download Chapter 4 Principle of Mathematical Induction Questions and Answers, Notes, 1st PUC Maths Question Bank with Answers, Karnataka State Board Solutions help you to revise complete Syllabus and score more marks in your examinations.

Karnataka 1st PUC Maths Question Bank Chapter 4 Principle of Mathematical Induction

Question 1.
Using the principle of mathematical induction, prove that
\(\text { (1) } 1+2+3+\cdots+n=\frac{n(n+1)}{2}\), for all n∈N
Answer:
1st PUC Maths Question Bank Chapter 4 Principle of Mathematical Induction 1
which is P(k +1)
Thus, P(k)⇒ P(k +1).
Hence, by mathematical induction P(n) is true for all n∈N

KSEEB Solutions

Question 2.
1 + 3 + 5 + 7 + …….. + (2n – 1) = n2, for all n∈N
Answer:
Let P(n):1 + 3 + 5 + 7+……….. +(2n-1) = n2
For n = 1, LHS = 1, RHS = 12 = 1
∴ LHS = RHS
∴ F(1) is true.
Let upwards assume P(k) is true for some k∈N
i.e., 1 + 3 + 5 + …. + (2K- 1) = kz
Adding (k + 1)th term = 2K + 1, on both sides, we get,
1 + 3 + 5 +……. +(2k -1)(2k + l) = k2 + 2k + 1
= (k +1)2
which is P(k +1)
Thus, P(k)⇒ P(k +1).
Hence, by mathematical induction P(n) is true for all n∈N

Question 3.
\(1^{2}+2^{2}+3^{2}+\ldots \ldots+n^{2}=\frac{n(n+1)(2 n+1)}{6}\)
Answer:
1st PUC Maths Question Bank Chapter 4 Principle of Mathematical Induction 2
1st PUC Maths Question Bank Chapter 4 Principle of Mathematical Induction 3
which is P(k +1)
Thus, P(k) ⇒ P(k +1).
Hence, by mathematical induction P(n) is true for all n∈N

KSEEB Solutions

Question 4.
\(1^{3}+2^{3}+3^{3}+\ldots \ldots+n^{3}=\left[\frac{n(n+1)}{2}\right]^{2} \) for all n∈N
Answer:
1st PUC Maths Question Bank Chapter 4 Principle of Mathematical Induction 4
which is P(k +1)
Thus, P(k) ⇒ P(k +1).
Hence, by mathematical induction P(n) is true for all n∈N

Question 5.
\(1 \cdot 2+2 \cdot 3+\dots+n(n+1)=\frac{n(n+1)(n+2)}{3}\) for all n∈N
Answer:
1st PUC Maths Question Bank Chapter 4 Principle of Mathematical Induction 5
which is P(k +1)
Thus, P(k) ⇒ P(k +1).
Hence, by mathematical induction P(n) is true for all n∈N

Question 6.
\( \begin{aligned} &1 \cdot 2 \cdot 3+2 \cdot 3 \cdot 4+\dots+n(n+1)(n+2)&=\frac{n(n+1)(n+2)(n+3)}{4},
\text { for all } n \in N \end{aligned}\)
Answer:
1st PUC Maths Question Bank Chapter 4 Principle of Mathematical Induction 6
1st PUC Maths Question Bank Chapter 4 Principle of Mathematical Induction 7
which is P(k +1)
Thus, P(k) ⇒ P(k +1).
Hence, by mathematical induction P(n) is true for all n∈N

Question 7.
\( \cfrac{1}{1 \cdot 2}+\cfrac{1}{2 \cdot 3}+\cfrac{1}{3 \cdot 4}+\dots+\cfrac{1}{n(n+1)}=\cfrac{n}{n+1}\) for all n∈N
Answer:
1st PUC Maths Question Bank Chapter 4 Principle of Mathematical Induction 8
1st PUC Maths Question Bank Chapter 4 Principle of Mathematical Induction 9
which is P(k +1)
Thus, P(k) ⇒ P(k +1).
Hence, by mathematical induction P(n) is true for all n∈N

Question 8.
\(\begin{aligned} &\cfrac{1}{1 \cdot 2 \cdot 3}+\cfrac{1}{2 \cdot 3 \cdot 4}+\cfrac{1}{3 \cdot 4 \cdot 5}+\dots+\cfrac{1}{n(n+1)(n+2)}&=\cfrac{n(n+3)}{4(n+1)(n+2)}, \text { for all } n \in N \end{aligned}\)
Answer:
1st PUC Maths Question Bank Chapter 4 Principle of Mathematical Induction 10
1st PUC Maths Question Bank Chapter 4 Principle of Mathematical Induction 11
which is P(k +1)
Thus, P(k) ⇒ P(k +1).
Hence, by mathematical induction P(n) is true for all n∈N

KSEEB Solutions

Question 9.
\( \begin{aligned}&\frac{1}{2 \cdot 5}+\frac{1}{5 \cdot 8}+\frac{1}{8 \cdot 11}+\cdots \cdot+\\ &\frac{1}{(3 n-1)(3 n+2)}=\frac{n}{6 n+4} \end{aligned}\)
Answer:
1st PUC Maths Question Bank Chapter 4 Principle of Mathematical Induction 12
1st PUC Maths Question Bank Chapter 4 Principle of Mathematical Induction 13
1st PUC Maths Question Bank Chapter 4 Principle of Mathematical Induction 14
which is P(k +1)
Thus, P(k) ⇒ P(k +1).
Hence, by mathematical induction P(n) is true for all n∈N

Question 10.
\(\begin{aligned} &\cfrac{1}{1 \cdot 4}+\cfrac{1}{4 \cdot 7}+\cfrac{1}{7 \cdot 10}+\cdots+\\ &\cfrac{1}{(3 n-2)(3 n+1)}=\cfrac{n}{3 n+1} \end{aligned} \)
Answer:
1st PUC Maths Question Bank Chapter 4 Principle of Mathematical Induction 15
1st PUC Maths Question Bank Chapter 4 Principle of Mathematical Induction 16
which is P(k +1)
Thus, P(k) ⇒ P(k +1).
Hence, by mathematical induction P(n) is true for all n∈N

Question 11.
\(\begin{aligned} &\cfrac{1}{3 \cdot 5}+\cfrac{1}{5 \cdot 7}+\cfrac{1}{7 \cdot 9}+\cdots+\\ &\cfrac{1}{(2 n+1)(2 n+3)}=\cfrac{n}{3(2 n+3)} \end{aligned}\)
Answer:
1st PUC Maths Question Bank Chapter 4 Principle of Mathematical Induction 17

1st PUC Maths Question Bank Chapter 4 Principle of Mathematical Induction 18
which is P(k +1)
Thus, P(k) ⇒ P(k +1).
Hence, by mathematical induction P(n) is true for all n∈N

Question 12.
\(\begin{aligned} &1+\cfrac{1}{1+2}+\cfrac{1}{1+2+3}+\dots+\\ &\cfrac{1}{1+2+3+\dots+n}=\cfrac{2 n}{n+1} \end{aligned} \)
Answer:
1st PUC Maths Question Bank Chapter 4 Principle of Mathematical Induction 20
1st PUC Maths Question Bank Chapter 4 Principle of Mathematical Induction 21
which is P(k +1)
Thus, P(k) ⇒ P(k +1).
Hence, by mathematical induction P(n) is true for all n∈N

KSEEB Solutions

Question 13.
\(\begin{aligned}&1 \cdot 3+3 \cdot 5+5 \cdot 7+\dots+(2 n-1)(2 n+1)&=\cfrac{n\left(4 n^{2}+6 n-1\right)}{3}, \text { for all } n \in N \end{aligned}\)
Answer:
1st PUC Maths Question Bank Chapter 4 Principle of Mathematical Induction 22
1st PUC Maths Question Bank Chapter 4 Principle of Mathematical Induction 23
which is P(k +1)
Thus, P(k) ⇒ P(k +1).
Hence, by mathematical induction P(n) is true for all n∈N

Question 14.
\(1+3+3^{2}+\cdots+3^{n-1}=\cfrac{3^{n}-1}{2} \)
Answer:
1st PUC Maths Question Bank Chapter 4 Principle of Mathematical Induction 24
1st PUC Maths Question Bank Chapter 4 Principle of Mathematical Induction 25
which is P(k +1)
Thus, P(k) ⇒ P(k +1).
Hence, by mathematical induction P(n) is true for all n∈N

Question 15.
\(\cfrac{1}{2}+\cfrac{1}{4}+\cfrac{1}{8}+\dots+\cfrac{1}{2^{n}}=1-\cfrac{1}{2^{n}} \)
Answer:
1st PUC Maths Question Bank Chapter 4 Principle of Mathematical Induction 26
which is P(k +1)
Thus, P(k) ⇒ P(k +1).
Hence, by mathematical induction P(n) is true for all n∈N

Question 16.
\( a+a r+a r^{2}+\cdots+a r^{n-1}=\cfrac{a\left(r^{n}-1\right)}{r-1}\)
Answer:
1st PUC Maths Question Bank Chapter 4 Principle of Mathematical Induction 27
1st PUC Maths Question Bank Chapter 4 Principle of Mathematical Induction 28
which is P(k +1)
Thus, P(k) ⇒ P(k +1).
Hence, by mathematical induction P(n) is true for all n∈N

Question 17.
\( 1 \cdot 2+2 \cdot 2^{2}+3 \cdot 2^{3}+\cdots+n 2^{n}=(n-1) 2^{n+1}+2\)
Answer:
1st PUC Maths Question Bank Chapter 4 Principle of Mathematical Induction 29
which is P(k +1)
Thus, P(k) ⇒ P(k +1).
Hence, by mathematical induction P(n) is true for all n∈N

Question 18.
\( \begin{aligned} &1 \cdot 3+2 \cdot 3^{2}+3 \cdot 3^{2}+\cdots+n \cdot 3^{n}&=\cfrac{(2 n-1) 3^{n+1}+3}{4}, \text { for all } n \in N \end{aligned} \)
Answer:
1st PUC Maths Question Bank Chapter 4 Principle of Mathematical Induction 30
1st PUC Maths Question Bank Chapter 4 Principle of Mathematical Induction 31
which is P(k +1)
Thus, P(k) ⇒ P(k +1).
Hence, by mathematical induction P(n) is true for all n∈N

KSEEB Solutions

Question 19.
\(\left(1+\frac{1}{1}\right)\left(1+\frac{1}{2}\right)\left(1+\frac{1}{3}\right) \cdots\left(1+\frac{1}{n}\right)=n+1\) for all n∈N
Answer:
1st PUC Maths Question Bank Chapter 4 Principle of Mathematical Induction 32
1st PUC Maths Question Bank Chapter 4 Principle of Mathematical Induction 33
which is P(k +1)
Thus, P(k) ⇒ P(k +1).
Hence, by mathematical induction P(n) is true for all n∈N

Question 20.
\( \left(1+\cfrac{3}{1}\right)\left(1+\cfrac{5}{4}\right)\left(1+\cfrac{7}{9}\right) \cdots\left(1+\cfrac{2 n+1}{n^{2}}\right)=(n+1)^{2}\)
Answer:
1st PUC Maths Question Bank Chapter 4 Principle of Mathematical Induction 34
which is P(k +1)
Thus, P(k) ⇒ P(k +1).
Hence, by mathematical induction P(n) is true for all n∈N

Question 21.
\(1^{2}+3^{2}+5^{2}+\cdots \cdot \cdot+(2 n-1)^{2}=\cfrac{n(2 n-1)(2 n+1)}{3}\)
Answer:
1st PUC Maths Question Bank Chapter 4 Principle of Mathematical Induction 35
1st PUC Maths Question Bank Chapter 4 Principle of Mathematical Induction 36
which is P(k +1)
Thus, P(k) ⇒ P(k +1).
Hence, by mathematical induction P(n) is true for all n∈N

Question 22.
Prove that 2n> n for all positives integers
Answer:
1st PUC Maths Question Bank Chapter 4 Principle of Mathematical Induction 37
Thus, P(k) ⇒ P(k +1).
Hence, by mathematical induction P(n) is true for all n∈N

Question 23.
\( 1+2+3 \cdots+n<\frac{1}{8}(2 n+1)^{2}\)
Answer:
1st PUC Maths Question Bank Chapter 4 Principle of Mathematical Induction 38
1st PUC Maths Question Bank Chapter 4 Principle of Mathematical Induction 39
which is P(k +1)
Thus, P(k) ⇒ P(k +1).
Hence, by mathematical induction P(n) is true for all n∈N

KSEEB Solutions

Question 24.
\(1^{2}+2^{2}+3^{2}+\cdots+n^{2}>\frac{n^{3}}{3}, n \in N \)
Answer:
1st PUC Maths Question Bank Chapter 4 Principle of Mathematical Induction 40
which is P(k +1)
Thus, P(k) ⇒ P(k +1).
Hence, by mathematical induction P(n) is true for all n∈N

Question 25.
(2n + 7) < (n + 3)2
Answer:
Let P(n): (2n + 7) < (n + 3)2
For = 1, (2 + 7)<(1+ 3)2 ⇒9<16 is true.
∴ P(1) is true.
Let us assume P(k) is true for some k∈ N
i.e., (2k+ 1) < (k + 3)2
Consider 2 (k + 1) + 7 = 2k + 2 + 7
= (2k + 7) + 2
< (k: + 3)2 + 2 = k2 + 6k + 9 + 2
<(k + 4)2
∴ P(k +1) is true.
Hence, by mathematical induction, P(n) is true for all n∈N

Question 26.
72n – 32n is divisible by 4.
Answer:
Let P(n): 7n – 3n is divisible by 4.
For n = 1, P(1): 71 – 31 = 4 which is divisible by 4.
∴ P(1) is true.
Let us assume P(k) is true for some k∈N
i.e., 72n – 32n is divisible by 4.
Thus, P(k) ⇒ P(k +1).
Hence, by mathematical induction P(n) is true for all n∈N

Question 27.
x2n – y2n is divisible by x+y.
Answer:
Let P(n): x2n – y2n is divisible by x + y
For n = 1, P(1): x2 – y2 is divisible by x + y, which is true.
∴ P(1) is true.
Let us assume P(k) is true for some k∈ N
i.e., x2k – y2k is divisible by x + y
Let x2k – y2k = (x + y)d, where d∈N
1st PUC Maths Question Bank Chapter 4 Principle of Mathematical Induction 41
which is P(k +1)
Thus, P(k) ⇒ P(k +1).
Hence, by mathematical induction P(n) is true for all n∈N

Question 28.
102n-1 +1 is divisible by
Answer:
Let P(n): 102n-1 +1, is divisible by 11.
For n = 1, P(1):102-1 +1, is divisible by 11, which is true.
P(1) is true.
Let us assume P(k) is true for some k∈N.
i.e., 102n-1+1 is divisible by 11.
⇒102-1 +1 = 1 W, d∈N.
Consider 102(k+1)-1+1 = 102k+1 +1
= 102k-1 102+1 = (11d-1)102+1
= 11d(102)-100 + 1 = 11(100d)-99
= 11(100d-9)
∴ 102(k+1)-1 +1 is divisible by 11.
Thus, P(k) ⇒ P(k +1)
Hence, by mathematical induction, P(n) is true for all n∈N

Question 29.
32n+2 -8n-9 is divisible by 8.
Answer:
Let P(n): 32n+2 -8n-9 is divisible by 8.
For n = 1, P(1): 32+2 – 8(2) – 9 = 64 is divisible by 8, which is true.
P( 1) is true.
Let us assume P(k) is true for some k∈N
i.e., 32k+2 -8k -9 is divisible by 8.
Let 32k+2-8k-9 = 8d, d∈N ………………(1)
Consider 32(k+1)+2 – 8(k +1) – 9 = 32k+4 – 8k – 8 – 9
= 32k+2 -32-8(k-1) = (8d + 8k + 9)9-8k-17 using (1)
= (8d + 8k)9 + 81-8k-17
= 8(d + k)9 + 64-8k = 8[9(d + k) + 8-k]
which is divisible by 8.
Thus, P(k) ⇒ P(k +1)
Hence, by mathematical induction, P(n) is true for all n∈N

KSEEB Solutions

Question 30.
2.7n + 3.5n – 5 is divisible by 24 for all n∈N
Answer:
Let P(n): 2.7n + 3.5n – 5 is divisible by 24.
For n = 1; P(1): 2.7n + 3.5n – 5 = 24 is divisible by 24.              ‘
∴ P(1) is true.
Let us assume, P(k) is true for some k∈N
i.e P(n): 2.7k + 3.5k – 5 is divisible by 24.
Let 2.7k + 3.5k – 5=24d,where d∈N
1st PUC Maths Question Bank Chapter 4 Principle of Mathematical Induction 42
Thus, P(k)⇒P(k+ l)
Hence, by mathematical induction, P(n) is true for all n∈N

Question 31.
41n -14n  is a multiple of 27.
Answer:
Let P(n): 41n -14n is a multiple of 27.
For n = 1, P(1): 41′-14′ = 47 is a multiple of 27.
∴ P(1) is true.
Let us assume P(k) is true for some k∈N
1st PUC Maths Question Bank Chapter 4 Principle of Mathematical Induction 43
Thus, P(k)⇒P(k+ 1)
Hence, by mathematical induction, P(n) is true for all n∈N

Question 32.
n(n +1)(n + 5) is a multiple of 3.
Answer:
Let P(n): n(n + 1)(n + 5) is a multiple of 3.
For n = 1, P(1): 1(1 +1)(1 + 5) = 12 is a multiple of 3.
∴ P(1) is true.
Let us assume P(k) is true for some n∈N
i.e., P(k):k(k + 1)(k + 5) is a multiple of 3.
Let k(k + 1)(k + 5) = 3d
Consider (k + 1)(k + 2)(k + 6)
= (k + 2)(k + 1)(k + 5 + 1)
= k(k + 1)(k + 5 + 1) + 2(k + 1)(k + 5 + 1)
= k(k +1 )(k + 5) + k(k +1) + 2(k +1 )(k + 6)

Question 33.
\((a b)^{n}=a^{n} b^{n}\)
Answer:
1st PUC Maths Question Bank Chapter 4 Principle of Mathematical Induction 44

KSEEB Solutions

Question 34.
(1 + x)n ≥ (1 + nx)for all natural number n, where x>-l.
Answer:
Let P(n): (1 + x)n ≥ (1 + nx), for x >-1
For n = 1; P(1): (1 + x)1 ≥ (1 + x) which is true.
Let us assume P(k) is true for some k∈N
i.e., (1 + x)k ≥ (1 + kx)
Consider (1 + x)k+1 = (1 + x)k (1 + x)
≥ (1 + kx)(1 + x)
= (l + kx + x + kx2)
≥(1 + (k + 1)x)
∵ kx2≥0.
which is P(k + 1)
Thus, P(k) ⇒ P(k +1)
Hence, by mathematical induction, P(n) is true for all n∈N